2010 Level II Mock Exam: Morning Session

Answer: B. “CFA Institute Research Objectivity Standards” (AIMR, 2003; adapted 2005) ..... expect to see periodic certification by Alleghany that it is in compliance with all of the ... Calculate and interpret the key financial ratios used by credit analysts. A is correct ... the portfolio that is described in the same table? A. Scenario ...
536KB taille 285 téléchargements 753 vues
2010 Level II Mock Exam: Morning Session ANSWERS AND REFERENCES

Theresa Lecompte Case Scenario Theresa Lecompte, CFA, is an equity analyst for Topaz Group, a large full-service financial firm that offers insurance, investment banking, brokerage and investment management services. Topaz has adopted the CFA Institute Research Objectivity Standards to demonstrate their commitment to managing and fully disclosing conflicts of interest to all investors that have access to the firm’s research. Lecompte’s primary responsibility is to follow the information technology sector for the firm’s research department. She is working on two follow-up reports for NanoMem (NM) and UniFlash (UF). Topaz makes markets in both companies’ securities and Lecompte owns a small position in NM only. Lecompte has an excellent relationship with company officials at NM. In the past, LeCompte has made favorable recommendations regarding NM. In appreciation, she was invited to attend a company-sponsored event last December that was held at an exclusive ski resort overseas. NM paid all expenses relating to the trip and provided some excellent entertainment activities for the attendees. Lecompte disclosed this benefit to her supervisor at Topaz. Shortly thereafter, Topaz issued a secondary offering for NM. Lecompte believes that her excellent relationship with the firm played a large part in securing this business. However, Lecompte considers her relationship with UF to be contentious since company officials seem reluctant to share as much information with her as they have in the past. She believes the change in their behavior is a direct result of recent reports she has written on the company, which have been less than favorable. Prior to publication of her follow-up reports, Lecompte shares her report on NM in its entirety with top management at NM. She shares only the part of her report on UF that provides factual information with UF management. Lecompte’s compensation at Topaz includes an annual salary plus a bonus based on both the accuracy of her recommendations over time and the overall profitability of the company. Topaz makes public disclosure of the extent to which research analyst compensation in general is dependent upon the firm’s investment banking revenues. Following the release of her reports in early March, Lecompte is invited to appear on a television program to discuss her recommendations. During her appearance, she makes the following statements: 1. “My firm makes markets in the securities of both NanoMem and UniFlash and I currently own a position in NanoMem .” By accessing this mock exam, you agree to the following terms of use: This mock exam is provided to currently-registered CFA candidates. Candidates may view and print the exam for personal exam preparation only. The following activities are strictly prohibited and may result in disciplinary and/or legal action: accessing or permitting access by anyone other than currently-registered CFA candidates; copying, posting to any website, emailing, distributing and/or reprinting the mock exam for any purpose.

2. “Although, I just issued my report on UniFlash which reflected a neutral rating, I really believe a sell rating is more appropriate.” When she returns to her office the following day, Lecompte is informed by her supervisor that a company official at UF called to express his disappointment and anger regarding the negative remarks she had made about UF during her television appearance. Lecompte states that she believes her deteriorating relationship with UF will make it difficult to effectively cover the company in the future and recommends that Topaz discontinue coverage of UF immediately. 1. In sharing her research material with the subject companies, Lecompte most likely violated CFA Institute Research Objectivity Standards with respect to her report(s) on: A. UniFlash. B. NanoMem. C. Both NanoMem and UniFlash. Answer: B “CFA Institute Research Objectivity Standards” (AIMR, 2003; adapted 2005) 2010 Modular Level II, Vol. 1, pp. 168, 172 Study Session 1-4-b Critique company policies and practices related to research objectivity and distinguish between changes required and changes recommended for compliance with the Research Objectivity Standards. B is correct as Lecompte violated Standard 6, Relationships with Subject Companies, in sharing the full report with NanoMem. Sharing any section of a research report that might communicate the analyst’s proposed recommendation, rating, or price target is prohibited by the Standards. 2. With respect to the company-sponsored event that Lecompte attended, did she violate any CFA Institute Standards? A. Yes. B. No, because she disclosed it to her employer. C. No, because the Standards permit entertainment as long as it is business related. Answer: A “Guidance for Standards I-VII” 2010 Modular Level II, Vol. 1, pp. 21-25 “CFA Institute Research Objectivity Standards” (AIMR, 2003; adapted 2005) By accessing this mock exam, you agree to the following terms of use: This mock exam is provided to currently-registered CFA candidates. Candidates may view and print the exam for personal exam preparation only. The following activities are strictly prohibited and may result in disciplinary and/or legal action: accessing or permitting access by anyone other than currently-registered CFA candidates; copying, posting to any website, emailing, distributing and/or reprinting the mock exam for any purpose.

2010 Modular Level II, Vol. 1, p. 172 Study Session 1-2-a, 1-4-b Demonstrate a thorough knowledge of the Code of Ethics and Standards of Professional Conduct by applying the Code and Standards to specific situations. Critique company policies and practices related to research objectivity and distinguish between changes required and changes recommended for compliance with the Research Objectivity Standards. A is correct as Lecompte violated both the CFA Institute Standard I (B) relating to independence and objectivity and the CFA Institute Research Objectivity Standard 6, Relationships with Subject Companies, by allowing a corporate issuer to pay for her travel expenses. 3. Regarding Lecompte’s compensation structure is Topaz in violation of CFA Institute’s Research Objectively Standards? A. No. B. Yes, with respect to overall profitability of the firm. C. Yes, with respect to accuracy of her recommendations. Answer: A “CFA Institute Research Objectivity Standards” (AIMR, 2003; adapted 2005) 2010 Modular Level II, Vol. 1, pp. 168, 171 Study Session 1-4-b Critique company policies and practices related to research objectivity and distinguish between changes required and changes recommended for compliance with the Research Objectivity Standards. A is correct as Research Objectivity Standard 5 recommends analyst’s compensation be based on the accuracy of recommendations over time. In addition, compensation should not be directly inked to investment banking or other finance activities, which it is not in this case. 4. According to the CFA Institute Research Objectivity Standards, does the first statement Lecompte makes in her television appearance provide all the recommended disclosures relating to potential conflicts of interest? A. Yes. B. Only with respect to UniFlash. C. Only with respect to NanoMem. Answer: B “CFA Institute Research Objectivity Standards” (AIMR, 2003; adapted 2005) By accessing this mock exam, you agree to the following terms of use: This mock exam is provided to currently-registered CFA candidates. Candidates may view and print the exam for personal exam preparation only. The following activities are strictly prohibited and may result in disciplinary and/or legal action: accessing or permitting access by anyone other than currently-registered CFA candidates; copying, posting to any website, emailing, distributing and/or reprinting the mock exam for any purpose.

2010 Modular Level II, Vol. 1, pp. 167, 170 Study Session 1-4-b Critique company policies and practices related to research objectivity and distinguish between changes required and changes recommended for compliance with the Research Objectivity Standards. B is correct as Lecompte provided all the recommended disclosures relating to potential conflicts of interest with respect to UniFlash. She should have disclosed, “benefit received” from NanoMem with regard to the trip as required by Standard 2. 5. Does Lecompte’s second statement during her TV appearance comply with the CFA Institute Research Objectivity Standards recommended procedures? A. Yes. B. No, with regard to her rating system. C. No, with regard to personal investments Answer: B “CFA Institute Research Objectivity Standards” (AIMR, 2003; adapted 2005) 2010 Modular Level II, Vol. 1, p. 175 Study Session 1-4-b Critique company policies and practices related to research objectivity and distinguish between changes required and changes recommended for compliance with the Research Objectivity Standards. B is correct as the recommended procedures for compliance with the CFA Institute Research Objectivity Standard 11 states that firms should prohibit covered employees from communicating a rating or recommendation that is different from the current published rating or recommendation. 6. With respect to Lecompte’s coverage of UniFlash, according to the CFA Institute Standards, the least appropriate course of action for Topaz to take would be to: A. discontinue coverage. B. change assigned analyst. C. upgrade recommendation. Answer: C Guidance for Standards I-VII” 2010 Modular Level II, Vol. 1, pp. 21-25, 80-81 Study Session 1-2-b By accessing this mock exam, you agree to the following terms of use: This mock exam is provided to currently-registered CFA candidates. Candidates may view and print the exam for personal exam preparation only. The following activities are strictly prohibited and may result in disciplinary and/or legal action: accessing or permitting access by anyone other than currently-registered CFA candidates; copying, posting to any website, emailing, distributing and/or reprinting the mock exam for any purpose.

Demonstrate a thorough knowledge of the Code of Ethics and Standards of Professional Conduct by applying the Code and Standards to specific situations. C is correct because according to the CFA Institute Standards I (B), and V (A), members and candidates must exercise diligence, independence, objectivity, and thoroughness in analyzing investments, making investment recommendations, and taking investment actions. Changing a written recommendation to what a subject company desires is not acting diligently, independently, objectively, and/or thoroughly.

By accessing this mock exam, you agree to the following terms of use: This mock exam is provided to currently-registered CFA candidates. Candidates may view and print the exam for personal exam preparation only. The following activities are strictly prohibited and may result in disciplinary and/or legal action: accessing or permitting access by anyone other than currently-registered CFA candidates; copying, posting to any website, emailing, distributing and/or reprinting the mock exam for any purpose.

Erica Huang Case Scenario Erica Huang is a derivatives trading advisor for Eastern Funds Company with expertise in forward and futures markets and contracts. She helps Eastern’s portfolio managers to evaluate forward and futures contracts and to make appropriate decisions when the use of these derivatives is required. When working with the portfolio managers, who have varying levels of derivatives knowledge, Huang is asked for input on issues of both an analytical and a conceptual nature. Three managers have approached her with the situations described below. Some of her responses to the portfolio managers rely on the financial market information given in Exhibit 1. Exhibit 1 Financial Market Information U.S. Three-month (90 day) Annualized Risk-free Rate U.S. Continuously Compounded Six-month (180-day) Annualized Riskfree Rate Broad Equity Index Level Broad Equity Index Continuously Compounded Annualized Dividend Yield Japanese Three-month (90 day) Annualized Risk-free Rate Japanese Yen Spot Price

6.00% 5.83% 1,250.00 3.00% 1.00% ¥112.00/$

Manager A, an equity manager, has two requests: 1) Six months ago, to hedge against an expected decline in the value of a common stock of which he held 100,000 shares, he entered into a forward contract to sell the underlying stock at a price of $80. The forward contract has three months to expiration and the stock is currently trading at $75. He wants to know the value of his current position on a per share basis. 2) He expects equities to go up and would like to take a long position in a 180-day forward contract on the Broad Equity Index, which a dealer has priced at 1,285.88. He wants to know whether the forward contract is fairly priced. Manager B manages Eastern’s Global Fund whose shareholders have approved the use of derivatives for hedging purposes. Knowing that she will receive a yen dividend payment in 90 days, she wants to know at what forward price she can sell yen for dollars. She also wants to understand the risks, if any, of entering into a forward contract and asks the following question:

By accessing this mock exam, you agree to the following terms of use: This mock exam is provided to currently-registered CFA candidates. Candidates may view and print the exam for personal exam preparation only. The following activities are strictly prohibited and may result in disciplinary and/or legal action: accessing or permitting access by anyone other than currently-registered CFA candidates; copying, posting to any website, emailing, distributing and/or reprinting the mock exam for any purpose.

“If I agree to sell yen for dollars through a forward contract, am I guaranteed to be able to sell the number of yen at the price stated in the forward contract at its expiration?” Manager C is responsible for a commodity portfolio and asks Huang the following questions: 1) “Because of mark-to-market, does a futures contract always have zero value? 2) Do convenience yields impact futures prices?” 7. Using a 360-day year, the current value of Manager A’s short position in the stock forward contract is closest to: A. $2.73. B. $3.84. C. $5.00. Answer: B “Forward Markets and Contracts,” Don M. Chance 2010 Modular Level II, Vol. 6, pp. 18-24 Study Session 16-58-a Explain how the value of a forward contract is determined at initiation, during the life of the contract, and at expiration. The value of a long position in a forward contract at any time is: Vt = St – F(0,T)/(1 + r)(T – t) Where S = the underlying price F = the forward price r = the risk-free rate T = the time to expiration at contract initiation t = the time elapsed since initiation. Then, Vt = 75 – 80/(1.06)0.25 = –3.84, but this is the value to the long position. The value to the short position has the opposite sign and is $3.84. 8. Using a 365-day year, Huang’s most appropriate response to Manager A with regard to the Broad Equity Index forward contract is that the contract is: A. fairly priced. B. not fairly priced because the no-arbitrage price should be 1,267.57. C. not fairly priced because the no-arbitrage price should be 1,268.36. Answer: B “Forward Markets and Contracts,” Don M. Chance 2010 Modular Level II, Vol. 6, pp. 26-30 By accessing this mock exam, you agree to the following terms of use: This mock exam is provided to currently-registered CFA candidates. Candidates may view and print the exam for personal exam preparation only. The following activities are strictly prohibited and may result in disciplinary and/or legal action: accessing or permitting access by anyone other than currently-registered CFA candidates; copying, posting to any website, emailing, distributing and/or reprinting the mock exam for any purpose.

Study Session 16-58-b Calculate and interpret the price and the value of an equity forward contract, assuming dividends are paid either discretely or continuously. The formula for the price of a forward contract on an equity index is: F(0, T) = S0e-(δc)Te(rc)T Where F(0,T) = the price of a forward contract initiated at time 0 and expiring at time T S0 = the spot price of the underlying δc = the continuously compounded dividend yield rc = the continuously compounded interest rate T = 180/365 = 0.4932, which is the time to expiration of the contract in years. Then, F = 1250(e– (0.03)(0.4932))(e0.0583(0.4932)) = 1,267.57. The price of the forward contract of 1,285.88 is too high. 9. Using a 365-day year, the 90-day yen/dollar forward price should be closest to: A. ¥106.72/$US. B. ¥110.67/$US. C. ¥113.34/$US. Answer: B “Forward Markets and Contracts,” Don M. Chance 2010 Modular Level II, Vol. 6, pp. 38-43 Study Session 16-58-c Calculate and interpret the price and the value of 1) a forward contract on a fixed income security, 2) a forward rate agreement (FRA), and 3) a forward contract on a currency. The formula for the forward exchange rate is: F(0,T) = S0{(1+r)T/(1+rf)T} Where F(0,T) = the forward exchange rate of a forward contract initiated at time 0 and expiring at time T, S0 = the spot price, r = the domestic risk-free rate, and rf = the foreign risk-free rate. The formula assumes the currency quote is dollars per yen. If the quote is yen per dollar (as is the case here), then the forward price is S0{(1+rf)T/(1+r)T}, so F = 112(1.01/1.06)90/365 = ¥110.67/$. Note that the continuous formula, F= S0erf Te–rT, can be used. Converting the given rates to continuous rates gives rf = ln(1.01) = 0.00995 and r = ln(1.06) = 0.05827. F = 112e(90/365)(0.00995–0.05827) = ¥110.67/$.

By accessing this mock exam, you agree to the following terms of use: This mock exam is provided to currently-registered CFA candidates. Candidates may view and print the exam for personal exam preparation only. The following activities are strictly prohibited and may result in disciplinary and/or legal action: accessing or permitting access by anyone other than currently-registered CFA candidates; copying, posting to any website, emailing, distributing and/or reprinting the mock exam for any purpose.

10. Huang’s most appropriate response to Manager B’s question is no, because if the yen decreases in value compared to the dollar, Manager B: A. will terminate the forward contract early. B. faces the risk the other party will default at expiration. C. will pay a mark-to-market adjustment resulting in a higher overall cost. Answer: B “Forward Markets and Contracts,” Don M. Chance 2010 Modular Level II, Vol. 6, pp. 44-45 Study Session 16-58-d Evaluate credit risk in a forward contract, and explain how market value is a measure of the credit risk to a party in a forward contract. At expiration, if the market value of the contract is positive (Manager B sold the yen at a higher price than she could sell it at expiration), Manager B will only receive the agreed upon price if the other party does not default. 11. What is Huang’s most appropriate response to Manager C’s first question? A. Yes. B. No, a futures contract has a value based on the price change since its last mark-to-market. C. No, a futures contract has a value based on the price change expected prior to expiration. Answer: B “Futures Markets and Contracts,” Don M. Chance 2010 Modular Level II, Vol. 6, pp. 88-89 Study Session 16-59-a, b Explain why the futures price must converge to the spot price at expiration. Determine the value of a futures contract. The value of a futures contract before it has been marked to market can be greater or less than $0. The value is the gain or loss accumulated since the last marked to market. 12. Huang’s most appropriate response to Manager C’s second question is they will: A. increase the futures price. B. decrease the futures price. C. not impact the futures price. By accessing this mock exam, you agree to the following terms of use: This mock exam is provided to currently-registered CFA candidates. Candidates may view and print the exam for personal exam preparation only. The following activities are strictly prohibited and may result in disciplinary and/or legal action: accessing or permitting access by anyone other than currently-registered CFA candidates; copying, posting to any website, emailing, distributing and/or reprinting the mock exam for any purpose.

Answer: B “Futures Markets and Contracts,” Don M. Chance 2010 Modular Level II, Vol. 6, pp. 95-98 Study Session 16-59-d Describe the monetary and non-monetary benefits and costs associated with holding the underlying asset, and explain how they affect the futures price. The futures price formula is f0(T) = So (1+r)T + FV(CB,0,T) where FV(CB,0,T) represents the future value of the costs of storage minus the convenience yield. Thus the convenience yield decreases the futures price.

By accessing this mock exam, you agree to the following terms of use: This mock exam is provided to currently-registered CFA candidates. Candidates may view and print the exam for personal exam preparation only. The following activities are strictly prohibited and may result in disciplinary and/or legal action: accessing or permitting access by anyone other than currently-registered CFA candidates; copying, posting to any website, emailing, distributing and/or reprinting the mock exam for any purpose.

Yeongsan Securities Case Scenario Hee-young Park, CFA is a senior portfolio manager at Yeongsan Securities, a Korean investment management firm. She manages the firm’s U.S. fixed income investments. Yeongsan owns $15 million in Alleghany Manufacturing Corp bonds. In preparation for analyzing the credit quality of the Alleghany bonds, Park has gathered financial data for the firm for 2007-2009, which is presented in Exhibit 1 below. Exhibit 1 Alleghany Financial Data (millions of US$) 2009 2008 Revenue 1,245 1,174 COGS 641 638 Selling & Administrative 256 231 Research & Development 41 17 Depreciation 82 97 EBIT 225 191 Interest Expense 97 86 EBT 128 105 Taxes 51 30 Net Income 77 75

2007 1,149 672 195 0 122 160 83 77 42 35

Total Assets Short-term Liabilities Long-term Liabilities Shareholder's Equity

2,185 112 1,426 647

2,310 189 1,287 834

2,243 147 1,368 728

The bonds are currently rated BB by Standard & Poor’s. Park knows that the major credit rating agencies consider many aspects of Alleghany’s financial condition and operations when determining its credit rating. In particular, Park believes that the ratings agencies will: Factor 1:

focus on Alleghany’s cash flows from investing, rather than its cash flows from financing activities, as the best measure of its ability to service debt.

Factor 2:

look favorably on the fact that most of Alleghany’s senior management compensation is performance based.

By accessing this mock exam, you agree to the following terms of use: This mock exam is provided to currently-registered CFA candidates. Candidates may view and print the exam for personal exam preparation only. The following activities are strictly prohibited and may result in disciplinary and/or legal action: accessing or permitting access by anyone other than currently-registered CFA candidates; copying, posting to any website, emailing, distributing and/or reprinting the mock exam for any purpose.

Factor 3:

expect to see periodic certification by Alleghany that it is in compliance with all of the bond’s covenants.

Park is evaluating a number of other securities for possible purchase. The bonds being considered include callable and non-callable corporate bonds with the same credit rating. Park wants a measure of how cheap or rich each security is relative to the others. In addition to managing credit risk, Park is responsible for managing the interest rate risk of her portfolios. She measures the interest rate risk of each portfolio for four maturities; 2, 5, 10, and 20 years. Each portfolio’s interest rate sensitivity is measured for a variety of yield curve change scenarios. Exhibit 2 below shows one portfolio’s dollar exposure to the four maturities, the key rate duration of maturity, the current yield at each maturity, and three yield curve scenarios. Exhibit 2 Key Rate Exposure, Current Yield Curve and Yield Curve Scenarios Maturity (years) 2 5 10 20 Key rate durations (years) 2 5 10 20 Portfolio key rate exposure ($millions) 60 30 15 40 Current yield curve 3.20% 3.85% 4.30% 4.75% Yield curve scenario 1 +0.50% +0.50% +0.50% +0.50% Yield curve scenario 2 +0.25% +0.50% +0.75% +1.00% Yield curve scenario 3 +1.00% +0.75% +0.50% +0.25%

13. Alleghany’s pretax return on capital indicates that its financial performance has most likely: A. improved. B. worsened. C. stayed the same. Answer: A “General Principles of Credit Analysis,” Frank J. Fabozzi 2010 Modular Level II, Vol. 5, pp. 171-174 and 205-210 (case study) Study Session 14-51-c Calculate and interpret the key financial ratios used by credit analysts. A is correct because pretax return on capital = EBIT/average of beginning and end of year capital. For 2009, pretax return on capital = 22.5/[(231.0+224.3)/2] = 9.9%. For 2008, pretax return on capital = 19.1/[(224.3+218.5)/2] = 8.6%. Higher returns indicate improved financial performance. By accessing this mock exam, you agree to the following terms of use: This mock exam is provided to currently-registered CFA candidates. Candidates may view and print the exam for personal exam preparation only. The following activities are strictly prohibited and may result in disciplinary and/or legal action: accessing or permitting access by anyone other than currently-registered CFA candidates; copying, posting to any website, emailing, distributing and/or reprinting the mock exam for any purpose.

14. Which of these ratios suggest Alleghany’s credit worthiness improved from 2007 to 2009? A. Operating income/sales B. EBITDA interest coverage C. Long-term debt/capitalization Answer: C “General Principles of Credit Analysis,” Frank J. Fabozzi 2010 Modular Level II, Vol. 5, pp. 171-174 and 205-210 (case study) Study Session 14-51-c Calculate and interpret the key financial ratios used by credit analysts. C is correct because Operating income/sales = (Sales – COGS – SG&A expense – R&D expense)/Sales EBITDA interest coverage = (EBIT + Depreciation)/Interest Expense Long-term debt/capitalization = Long-term Debt/(Long-term Debt + Shareholders Equity) The table below provides these ratios for 2007-2009

Operating income/sales EBITDA interest coverage LTD/Capitalization

2009 0.25 3.16 0.61

2008 0.25 3.35 0.65

2007 0.25 3.40 0.69

Operating income/sales is flat. EBITDA interest coverage has decreased, indicating worsening financial performance. Long-term debt/capitalization has decreased indicating improved financial condition. 15. Of the factors Park believes will be important in the rating of Alleghany’s bonds, which is most likely incorrect? A. Factor 1. B. Factor 2. C. Factor 3. Answer: A “General Principles of Credit Analysis,” Frank J. Fabozzi By accessing this mock exam, you agree to the following terms of use: This mock exam is provided to currently-registered CFA candidates. Candidates may view and print the exam for personal exam preparation only. The following activities are strictly prohibited and may result in disciplinary and/or legal action: accessing or permitting access by anyone other than currently-registered CFA candidates; copying, posting to any website, emailing, distributing and/or reprinting the mock exam for any purpose.

2010 Modular Level II, Vol. 5, pp. 176-178 and 182 Study Session 14-51-b Explain and analyze the key components of credit analysis. A is correct because Alleghany’s credit rating implies that given its weak financial condition as a speculative grade credit, there is heightened focus on free cash flow. Speculative grade issuers typically face near term vulnerabilities. Cash flow from operations would be a starting point for assessing the company’s ability to service obligations through its operating activities, followed by additional consideration to cash flow from investing and financing activities. (p. 177). 16. The measure of security richness or cheapness that best meets Park’s requirements is the: A. nominal spread. B. zero-volatility spread. C. option-adjusted spread. Answer: C “Valuing Bonds with Embedded Options,” Frank J. Fabozzi 2010 Modular Level II, Vol. 5, pp. 295-299 Study Session 14-54-b Evaluate the importance of benchmark interest rates in interpreting spread measures. C is correct because Park is evaluating both callable and noncallable bonds and therefore requires a measure that does not include reflect the value of embedded options, which requires an option adjusted spread. 17. Which of the yield curve scenarios listed in Exhibit 2 will cause the largest loss in the portfolio that is described in the same table? A. Scenario 1 B. Scenario 2 C. Scenario 3 Answer: B “Term Structure and Volatility of Interest Rates,” Frank J. Fabozzi 2010 Modular Level II, Vol. 5, pp. 256-258 Study Session 14-53-f Compute and interpret the yield curve risk of a security or a portfolio by using key rate duration. By accessing this mock exam, you agree to the following terms of use: This mock exam is provided to currently-registered CFA candidates. Candidates may view and print the exam for personal exam preparation only. The following activities are strictly prohibited and may result in disciplinary and/or legal action: accessing or permitting access by anyone other than currently-registered CFA candidates; copying, posting to any website, emailing, distributing and/or reprinting the mock exam for any purpose.

B is correct. The expected loss of a portfolio approximately equals the sum, across the key rate maturities, of the dollar exposure multiplied by the key rate duration multiplied by the increase in the key rate. The losses the portfolio would experience for each yield curve scenario are: Scenario 1: 60 0.005 2 30 0.005 5 15 0.005 10 40 0.005 20 6.1 Scenario 2: 60 0.0025 2 30 0.005 5 15 0.0075 10 40 0.01 20 10.2 Scenario 3: 60 0.01 2 30 0.0075 5 15 0.005 10 40 0.0025 20 5.1 18. Yield curve scenario 3 in Exhibit 2 is best described as an example of: A. a flattening of the curve. B. a positive butterfly shift. C. an upward parallel shift. Answer: A “Term Structure and Volatility of Interest Rates,” Frank J. Fabozzi 2010 Modular Level II, Vol. 5, pp. 239-240 Study Session 14-53-a Illustrate and explain parallel and nonparallel shifts in the yield curve, a yield curve twist, and a change in the curvature of the yield curve (i.e., a butterfly shift.) A is correct. The changes in scenario 3 get progressively smaller as the maturity increases, so the shift is neither parallel (equal change at all maturities) or a butterfly shift (change for shortest and longest maturities is systematically larger or smaller than the change for the intermediate maturities.) If we add the changes to the existing yield curve, the difference between the 20-year yield and the 2-year yield (the slope) decreases from 1.55% to 0.80%, indicating a flattening of the curve.

By accessing this mock exam, you agree to the following terms of use: This mock exam is provided to currently-registered CFA candidates. Candidates may view and print the exam for personal exam preparation only. The following activities are strictly prohibited and may result in disciplinary and/or legal action: accessing or permitting access by anyone other than currently-registered CFA candidates; copying, posting to any website, emailing, distributing and/or reprinting the mock exam for any purpose.

Joan Hammond Case Scenario Joan Hammond is the manager of Sparta Corporation’s pension fund. She recently presented her quarterly report to the fund’s board of directors, and is recommending a change in the fund’s asset allocation. Based on her market expectations, she recommends an allocation of 30 percent of assets to value stocks, 50 percent to growth stocks, and 20 percent to bonds. Hammond’s market expectations are shown in Exhibit 1. Exhibit 1 Hammond’s Market Expectations Value Stock Growth Stock Portfolio Portfolio Expected Annual Return (%) 12 14 Expected Standard Deviation of 16 22 Annual Returns (%) Return Correlations Value Stock Portfolio 1.0 0.9 Growth Stock Portfolio --1.0 Bond Portfolio -----

Bond Portfolio 8 8 0.3 0.2 1.0

Board member Benjamin Donner is skeptical about the recommended change in asset allocation for the pension fund, and he has several questions for Hammond. He asks Hammond to describe the risk and return characteristics of her recommended portfolio. Hammond responds: “I believe that the recommended asset allocation will produce a portfolio that is the global minimum-variance portfolio. The global minimum-variance portfolio has the lowest level of risk compared to all other portfolios on the efficient frontier and therefore it also dominates all other portfolios on the efficient frontier.” Donner argues that Hammond should consider broadening the diversification of the fund’s portfolio into a “fully diversified portfolio” by adding real estate and international stocks. He states that these additions will improve the efficiency of the fund. Donner estimates that the “fully diversified portfolio” would have an expected return of 13 percent and a standard deviation of 15 percent. He would then further expand the investment opportunity set by combining the proposed “fully diversified portfolio” with either risk-free borrowing or lending. He notes that the appropriate risk-free rate of return is 4 percent.

By accessing this mock exam, you agree to the following terms of use: This mock exam is provided to currently-registered CFA candidates. Candidates may view and print the exam for personal exam preparation only. The following activities are strictly prohibited and may result in disciplinary and/or legal action: accessing or permitting access by anyone other than currently-registered CFA candidates; copying, posting to any website, emailing, distributing and/or reprinting the mock exam for any purpose.

Finally, Donner recommends that Hammond include a fundamental factor model analysis in future reports. Donner states that fundamental factor models relate asset returns both to surprises in macroeconomic variables and to company attributes such as market capitalization. He believes such an analysis will be beneficial in making future asset allocation decisions. 19. Using Hammond’s recommended asset allocation and return expectations, the expected standard deviation of annual returns for the pension fund’s portfolio is closest to: A. 12.1%. B. 15.9%. C. 17.4%. Answer: B “Portfolio Concepts,” Richard A. Defusco, Dennis W. McLeavey, Jerald E. Pinto, and David E. Runkel 2010 Modular Level II, Vol. 6, pp. 374-377 Study Session 18-64-a Discuss mean-variance analysis and its assumptions, and calculate the expected return and the standard deviation of return for a portfolio of two or three assets. B is correct. SD = (0.32×16%2 + 0.52×22%2 + 0.22×8%2 + 2×0.3×0.5×16%×22%×0.9 + 2×0.3×0.2×16%×8%×0.3 + 2×0.5×0.2×22%× 8%×0.2)0.5 = 15.9%. 20. If Hammond wants to achieve an expected annual return of 12.5% while maintaining the pension fund’s current 20% allocation to bonds, the proportion of the fund’s assets that should be allocated to value stocks is closest to: A. 15%. B. 65%. C. 75%. Answer: A “Portfolio Concepts,” Richard A. Defusco, Dennis W. McLeavey, Jerald E. Pinto, and David E. Runkel 2010 Modular Level II, Vol. 6, pp. 374-377 Study Session 18-64-a Discuss mean-variance analysis and its assumptions, and calculate the expected return and the standard deviation of return for a portfolio of two or three assets.

By accessing this mock exam, you agree to the following terms of use: This mock exam is provided to currently-registered CFA candidates. Candidates may view and print the exam for personal exam preparation only. The following activities are strictly prohibited and may result in disciplinary and/or legal action: accessing or permitting access by anyone other than currently-registered CFA candidates; copying, posting to any website, emailing, distributing and/or reprinting the mock exam for any purpose.

A is correct. The equation to solve for the weights for growth and value stocks given a 20 percent allocation to bonds is: E(r) = w ×12% + (0.8 – w) ×14% + 0.2 × 8%, where w = % in value stocks and (0.8 – w) = % in growth stocks. Setting E(r) equal to 12.5% and solving: w = 0.15 and (0.8 – w) = 0.65. 21. Hammond’s statement about the risk level and dominance of the global minimum-variance is? A. correct about risk level and correct about dominance. B. correct about risk level but incorrect about dominance. C. incorrect about risk level but correct about dominance. Answer: B “Portfolio Concepts,” Richard A. Defusco, Dennis W. McLeavey, Jerald E. Pinto, and David E. Runkel 2010 Modular Level II, Vol. 6, pp. 377-381 Study Session 18-64-b Explain the minimum-variance and efficient frontiers, and discuss the steps to solve for the minimum-variance frontier. B is correct. While she is correct that the global minimum variance portfolio has the lowest level of risk compared to other portfolios on the efficient frontier, she is incorrect with regard to dominance. The global minimum variance portfolio does not dominate portfolios on the efficient frontier. 22. If Donner wants to construct an optimal portfolio that has an expected standard deviation of annual returns of 12percent, he should combine his proposed “fullydiversified portfolio” with which of the following actions? A. Lend 20% of total assets. B. Lend 80% of total assets. C. Borrow 20% of total assets. Answer: A “Portfolio Concepts,” Richard A. Defusco, Dennis W. McLeavey, Jerald E. Pinto, and David E. Runkel 2010 Modular Level II, Vol. 6, pp. 395-403 Study Session 18-64-d Calculate the variance of an equally-weighted portfolio of n stocks; explain the capital allocation and the capital market lines (CAL and CML), and the relation between them, and calculate the values of one of the variables given the values of the remaining variables. By accessing this mock exam, you agree to the following terms of use: This mock exam is provided to currently-registered CFA candidates. Candidates may view and print the exam for personal exam preparation only. The following activities are strictly prohibited and may result in disciplinary and/or legal action: accessing or permitting access by anyone other than currently-registered CFA candidates; copying, posting to any website, emailing, distributing and/or reprinting the mock exam for any purpose.

A is correct. The standard deviation of the portfolio is: p = w1 1; so 12% = w1 15%; w1 = 0.8 = weight in risky portfolio. The balance, 1 – 0.8 = 0.2 or 20%, should be invested in risk-free assets. 23. If Donner uses his proposed “fully-diversified portfolio” to construct an optimal portfolio that has an expected standard deviation of annual returns of 12 percent, the expected annual return for the resulting portfolio is closest to: A. 7.2%. B. 11.2%. C. 14.4%. Answer: B “Portfolio Concepts,” Richard A. Defusco, Dennis W. McLeavey, Jerald E. Pinto, and David E. Runkel 2010 Modular Level II, Vol. 6, pp. 395-403 Study Session 18-64-d Calculate the variance of an equally-weighted portfolio of n stocks; explain the capital allocation and the capital market lines (CAL and CML), and the relation between them, and calculate the values of one of the variables given the values of the remaining variables. B is correct. E(R) = RF + (E(Rp) – RF) ×σ/σp = 4% + [(13% – 4%) ×12%]/15% = 11.2%. This can also be obtained by: E(R) = (1 – wp) RF + wp Rp = (1– 0.80) 4% + 0.80 13% = 11.2%. The correct answer can also be obtained by: (1) Calculating weight of risk-free asset 12= (1-w ) x 15 => w=20% (w is the weight of the risk-free asset in the portfolio) (2) Then calculate the return with risk-free asset and risky-portfolio R=20% x 4 +80% x 13 =0.8+10.4=11.2% 24. Is Donner’s description of the factor model he recommends to Hammond correct? A. Yes. B. No, he is incorrect about surprises in macroeconomic variables and correct about company attributes. C. No, he is correct about surprises in macroeconomic variables and incorrect about company attributes. Answer: B By accessing this mock exam, you agree to the following terms of use: This mock exam is provided to currently-registered CFA candidates. Candidates may view and print the exam for personal exam preparation only. The following activities are strictly prohibited and may result in disciplinary and/or legal action: accessing or permitting access by anyone other than currently-registered CFA candidates; copying, posting to any website, emailing, distributing and/or reprinting the mock exam for any purpose.

“Portfolio Concepts,” Richard A. Defusco, Dennis W. McLeavey, Jerald E. Pinto, and David E. Runkel 2010 Modular Level II, Vol. 6, pp. 420-421 Study Session 18-64-j Discuss and compare macroeconomic factor models, fundamental factor models, and statistical factor models. B is correct. Donner’s statement about surprises in macroeconomic variables is incorrect because macroeconomic factor models, not fundamental factor models, relate asset returns to surprises in macroeconomic variables. His statement about company attributes is correct because fundamental factor models do relate asset returns to company attributes such as market capitalization.

By accessing this mock exam, you agree to the following terms of use: This mock exam is provided to currently-registered CFA candidates. Candidates may view and print the exam for personal exam preparation only. The following activities are strictly prohibited and may result in disciplinary and/or legal action: accessing or permitting access by anyone other than currently-registered CFA candidates; copying, posting to any website, emailing, distributing and/or reprinting the mock exam for any purpose.

Diotrephes Foundation Case Scenario Aydin Yusuf, CFA, is director of investments for the Diotrephes Foundation, a non-profit that supports Turkish artistic and cultural events in the U.S. Yusuf has lowered his return expectations for the portfolio’s equity and fixed income holdings for the next two years. In order to improve the portfolio’s return, he is considering allocations to additional asset classes, including alternative investments. His current focus is commodities and hedge funds. After a careful assessment of investing in commodities, Yusuf decides that he will seek investments in those commodities that are most likely to produce positive “roll yield.” Yusuf is also considering investments that will track a broad commodity index. He is confused, however, by reports that illustrate that while the long-run geometric return of the average commodity was close to zero, the geometric return of the commodity index for the same period was strongly positive. He wants to do further analysis into the cause of this apparent disparity. Yusuf focuses on establishing goals for hedge fund investments before making any investment decisions about specific hedge funds or hedge fund strategies. He decides that Diotrephes will benchmark hedge fund investments against the 30-day Treasury bill rate plus a spread of 250 bps. Yusuf considered other benchmarks, including hedge fund indexes. He found that there are numerous statistical problems associated with these indexes, including: Problem 1: backfill bias may overestimate returns. Problem 2: the funds in each index are subject to turnover. Problem 3: autocorrelation in returns may overestimate volatility. Yusuf understands that hedge fund investments will be substantially riskier than the portfolio’s investments in fixed income and equities. Yusuf decides to use maximum drawdown as a risk measure to compare the risks of the hedge funds that he is considering for the portfolio. After his analysis is completed, Yusuf identifies a multistrategy hedge fund that invests in fixed income, equities, and commodities. 25. The commodities in which Yusuf invests are most likely to have: A. few limitations to storage. B. prices that are volatile and at historic lows. C. prices that are volatile and at historic highs. Answer: B “Investing in Commodities,” Ronald G. Layard-Liesching 2010 Modular Level II, Vol. 5, pp. 116-120 Study Session 13-48-b By accessing this mock exam, you agree to the following terms of use: This mock exam is provided to currently-registered CFA candidates. Candidates may view and print the exam for personal exam preparation only. The following activities are strictly prohibited and may result in disciplinary and/or legal action: accessing or permitting access by anyone other than currently-registered CFA candidates; copying, posting to any website, emailing, distributing and/or reprinting the mock exam for any purpose.

Discuss how “roll yield” in a commodity futures price can be positive (negative). B is correct because positive “roll yield” occurs when the futures price is above the “full carry” price, referred to as backwardation, which may occur when prices are low and volatile and producers are concerned that they will fall further, to a level that is unprofitable. Producers will accept less that the “full carry” price in order to hedge price risk. 26. Yusuf’s confusion about commodity index returns is best explained by: A. misreporting of historical index values. B. changes in the index constituents over time. C. rebalancing of the index due to commodity price changes. Answer: C “Investing in Commodities,” Ronald G. Layard-Liesching 2010 Modular Level II, Vol. 5, p. 121 Study Session 13-48-d Demonstrate how the geometric return of an actively managed commodity basket can be positive, whereas the underlying average commodity has a geometric return near zero. C is correct because when prices of individual commodities change, the index must be rebalanced, with a decreased allocation to commodities whose prices increased and an increased allocation to those whose prices decreased. Booth and Fama (1992) showed that the geometric return of a rebalanced portfolio is higher than the average return of its constituents. 27. Yusuf’s intended benchmark for hedge fund investments would be most appropriate for: A. distressed securities funds. B. equity market neutral funds. C. fixed income arbitrage funds. Answer: B “Evaluating the Performance of Your Hedge Funds,” Vinh Q. Tran 2010 Modular Level II, Vol. 5, pp. 133-136 Study Session 13-49-b Compare and contrast the use of market indices, hedge fund indices, and positive risk-free rates to evaluate hedge fund performance.

By accessing this mock exam, you agree to the following terms of use: This mock exam is provided to currently-registered CFA candidates. Candidates may view and print the exam for personal exam preparation only. The following activities are strictly prohibited and may result in disciplinary and/or legal action: accessing or permitting access by anyone other than currently-registered CFA candidates; copying, posting to any website, emailing, distributing and/or reprinting the mock exam for any purpose.

B is correct because, while “application of some universal benchmark, no matter how well constructed, is unlikely to capture the essence of all hedge funds’ performances” (p. 136), “the hedge fund strategy that comes closest to pure riskfree arbitrage is equity market neutral.” (p. 135.) It is reasonable to benchmark risk-free arbitrage against the risk free rate plus a spread to represent return required to compensate for management fees. 28. Of the problems with hedge fund indices that Yusuf has identified, which is most likely incorrect? A. Problem 1 B. Problem 2 C. Problem 3 Answer: C “Evaluating the Performance of Your Hedge Funds,” Vinh Q. Tran 2010 Modular Level II, Vol. 5, pp. 134-135 Study Session 13-49-b Compare and contrast the use of market indices, hedge fund indices, and positive risk-free rates to evaluate hedge fund performance. C is correct because autocorrelation in hedge fund returns will underestimate volatility, not overestimate it. See table 2 on page 135. 29. The hedge fund risk measure that Yusuf selects: A. typically assumes that returns are normally distributed. B. measures the probability that a loss of a certain size will occur. C. focuses on the minimum value between successive maximum values. Answer: C “Buyers Beware: Evaluating and Managing the Many Faces of the Risks of Hedge Funds,” Vinh Q. Tran 2010 Modular Level II, Vol. 5, pp. 156-157 Study Session 13-50-b Evaluate maximum drawdown and value-at-risk for measuring risks of hedge funds. C is correct because Yusuf has chosen maximum drawdown as a risk measure which looks at the difference between peaks (maximums) and valleys (minimums) in the portfolio’s value and finds the largest such difference over a given period of time. By accessing this mock exam, you agree to the following terms of use: This mock exam is provided to currently-registered CFA candidates. Candidates may view and print the exam for personal exam preparation only. The following activities are strictly prohibited and may result in disciplinary and/or legal action: accessing or permitting access by anyone other than currently-registered CFA candidates; copying, posting to any website, emailing, distributing and/or reprinting the mock exam for any purpose.

30. The hedge fund Yusuf identifies is least likely to be at risk of: A. style drift. B. changes in credit spreads. C. rising correlations between equity and fixed income. Answer: A “Buyers Beware: Evaluating and Managing the Many Faces of the Risks of Hedge Funds,” Vinh Q. Tran 2010 Modular Level II, Vol. 5, p. 151 Study Session 13-50-a Discuss common types of investment risks for hedge funds. A is correct because the hedge fund is multistrategy and is expected to move between asset sectors and trading strategies.

By accessing this mock exam, you agree to the following terms of use: This mock exam is provided to currently-registered CFA candidates. Candidates may view and print the exam for personal exam preparation only. The following activities are strictly prohibited and may result in disciplinary and/or legal action: accessing or permitting access by anyone other than currently-registered CFA candidates; copying, posting to any website, emailing, distributing and/or reprinting the mock exam for any purpose.

Merick Manufacturing Case Scenario Merick Manufacturing is a U.S. based textile manufacturer whose equity securities are listed on the New York Stock Exchange and the London Stock Exchange. Merick prepares financial statements under both U.S. Generally Accepted Accounting Principles (U.S. GAAP) and International Financial Reporting Standards (IFRS). On 1 January 2009, Merick acquired 50 percent of the equity of Lisam, Inc., a small international textile manufacturer. The purchase price was $500 million in cash. The remaining 50 percent equity in Lisam is owned by a governmental entity outside of the U.S. Kim King, CFA, has been assigned the task of determining the potential effect of the acquisition on Merick’s reported financial results using both U.S. GAAP and IFRS. She is particularly interested in the potential effect on various financial ratios that might occur if the equity method is used. King is unsure at this point whether Merick will account for the acquisition using the equity method, proportionate consolidation method, or consolidation method. She has tentatively concluded, however, that under U.S. GAAP Merick is unlikely to have control over Lisam, but that under IFRS Merick is likely to be deemed to have joint control. Even if Merick uses the particular accounting methods prescribed by U.S. GAAP and IFRS, King may produce financial statements with alternative methods to improve financial information for users. King uses historical 31 December 2008 balance sheets (Exhibit 1) and projected income statements for the year ending 31 December 2009 (Exhibit 2) for Merick and Lisam to determine how the financial results and ratios may differ under each of the three acquisition accounting methods. The balance sheet was prepared immediately following the acquisition, but the projected income statements do not reflect the acquisition, and there are no inter-company transactions between Merick and Lisam. For ratio computation purposes, King uses beginning-of-year balance sheet values rather than average balance sheet values. She considers this appropriate because year-end projected balance sheets are expected to remain essentially unchanged for both firms other than the direct effects of the acquisition. The fair values of Lisam’s assets and liabilities at 31 December 2008 are equal to their historical reported amounts.

By accessing this mock exam, you agree to the following terms of use: This mock exam is provided to currently-registered CFA candidates. Candidates may view and print the exam for personal exam preparation only. The following activities are strictly prohibited and may result in disciplinary and/or legal action: accessing or permitting access by anyone other than currently-registered CFA candidates; copying, posting to any website, emailing, distributing and/or reprinting the mock exam for any purpose.

Exhibit 1 Balance Sheets on 31 December 2008 (Immediately Following Acquisition)

Cash Inventory Other Current Assets Total Current Assets Plant and Equipment Accumulated Depreciation Long-term investment Other Non-current Assets Total Assets Short-term Debt Other Current Liabilities Total Current Liabilities Long-term Debt Total Liabilities Common Stock Retained Earnings Shareholders’ Equity Total Liabilities and Shareholders’ Equity

Merick Manufacturing Common Size US$ Percentage Millions (%) $400 16.00 500 20.00 100 4.00 $1,000 40.00 1,400 56.00 (500) (20.00) 500 20.00 100 4.00 $2,500 100.00

Lisam, Inc.

US$ Millions $100 300 200 $600 800 (200)

Common Size Percentage (%) 6.67 20.00 13.33 40.00 53.33 (13.33)

300 $1,500

20.00 100.00

$200 200 $400 800 $1,200 200 1,100 $1,300

8.00 8.00 16.00 32.00 48.00 8.00 44.00 52.00

$0 200 $200 300 $500 100 900 $1,000

0.00 13.33 13.33 20.00 33.33 6.67 60.00 66.67

$2,500

100.00

$1,500

100.00

By accessing this mock exam, you agree to the following terms of use: This mock exam is provided to currently-registered CFA candidates. Candidates may view and print the exam for personal exam preparation only. The following activities are strictly prohibited and may result in disciplinary and/or legal action: accessing or permitting access by anyone other than currently-registered CFA candidates; copying, posting to any website, emailing, distributing and/or reprinting the mock exam for any purpose.

Exhibit 2 Projected Income Statements for the Year Ending 31 December 2009 Prior to the determination of Investment Income

Net Sales Cost of Goods Sold Gross Profit Depreciation Expense Other Expenses Operating Income Interest Expense Pretax Income Income Taxes Net Income

Merick Manufacturing Common Size US$ Percentage Millions (%) $3,750 100.00 (2,250) (60.00) 1,500 40.00 (140) (3.73) (985) (26.27) 375 10.00 (75) (2.00) 300 8.00 (120) (3.20) $180 4.80

Lisam, Inc.

US$ Millions $2,500 (1,600) 900 (60) (600) 240 (30) 210 (84) $126

Common Size Percentage (%) 100.00 (64.00) 36.00 (2.40) (24.00) 9.60 (1.20) 8.40 (3.36) 5.04

31. If King’s tentative conclusions about Merick’s control are correct, the equity method is most likely the preferred method for accounting for the acquisition under: A. IFRS only. B. U.S. GAAP only. C. both U.S. GAAP and IFRS. Answer: B “Intercorporate Investments,” Susan Perry Williams 2010 Modular Level II, Vol. 2, pp. 19-20, 32-33 Study Session 5-21-b Distinguish between IFRS and U.S. GAAP in the classification, measurement, and disclosure of investments in financial assets, investments in associates, joint ventures, business combinations, and special purpose and variable interest entities. Because Merick owns 50% (not more than 50%) of Lisam and does not control it, under U.S. GAAP the equity method is most appropriate. Because Merick is deemed to have joint control under IFRS, the proportionate consolidation method is preferred, although the equity method is acceptable under IFRS. (International standard setters are revisiting the issue of proportionate consolidation versus the equity method. These rules could change in the future.) By accessing this mock exam, you agree to the following terms of use: This mock exam is provided to currently-registered CFA candidates. Candidates may view and print the exam for personal exam preparation only. The following activities are strictly prohibited and may result in disciplinary and/or legal action: accessing or permitting access by anyone other than currently-registered CFA candidates; copying, posting to any website, emailing, distributing and/or reprinting the mock exam for any purpose.

32. Immediately after the acquisition, Merick’s current ratio would most likely be lowest under which method? A. Equity B. Consolidation C. Proportionate consolidation Answer: A “Intercorporate Investments,” Susan Perry Williams 2010 Modular Level II, Vol. 2, pp. 20-21, 33-35, 43 Study Session 5-21-c Analyze the effects on financial ratios of the different methods used to account for intercorporate investments. The balance sheet presented includes the investment in Lisam at cost, which, at the date of acquisition, would be the same as the equity method. Because Lisam’s current ratio is higher than Merick’s either of Consolidation or Proportionate Consolidation will increase Merick’s current ratio.

Current Assets Current Liabilities Current Ratio

Merick (Equity)

Lisam

Consolidation

$1,000 $400 2.5

$600 $200 3.0

1,600 600 2.67

Proportionate Consolidation (only 50% of Lisam added) 1,300 500 2.6

33. If Merick uses the preferred method for accounting for the acquisition under IFRS, the long-term debt to total asset ratio (%) would be closest to: A. 29. B. 32. C. 35. Answer: C “Intercorporate Investments,” Susan Perry Williams 2010 Modular Level II, Vol. 2, pp. 32-35 Study Session 5-21-a, c Describe the classification, measurement, and disclosure under the International Financial Reporting Standards (IFRS) for 1) investments in financial assets, 2) investments in associates, 3) joint ventures, 4) business combinations, and 5) special purpose and variable interest entities (SPEs and VIEs). Analyze the effects on financial ratios of the different methods used to account for intercorporate investments. By accessing this mock exam, you agree to the following terms of use: This mock exam is provided to currently-registered CFA candidates. Candidates may view and print the exam for personal exam preparation only. The following activities are strictly prohibited and may result in disciplinary and/or legal action: accessing or permitting access by anyone other than currently-registered CFA candidates; copying, posting to any website, emailing, distributing and/or reprinting the mock exam for any purpose.

The preferred method under IFRS is proportionate consolidation.

Merick (Equity)

Lisam

Proportionate Consolidation (only 50% of Lisam added) 800 + 150 = 950 2,500 – 500* + 750 = 2,750 35%

Long-term debt $800 $300 Total assets $2,500 $1,500 Long-term debt to 32% 20% total assets ratio *must deduct $500 investment when consolidating

34. After incorporating Lisam’s projected results Merick’s gross profit margin for the year ending 31 December 2009 will be most likely be highest under: A. Consolidation. B. The equity method. C. Proportionate consolidation. Answer: B “Intercorporate Investments,” Susan Perry Williams 2010 Modular Level II, Vol. 2, pp. 20, 33-35, 42- 47 Study Session 5-21-c Analyze the effects on financial ratios of the different methods used to account for intercorporate investments. The gross profit margin is highest under the equity method. Equity method:

Consolidation Method:

Proportionate consolidation method:

Merick’s share of Lisam’s earnings is reported on the income statement below the gross profit. Therefore, Merick’s sales and cost of goods sold would remain unchanged. Gross profit margin would be 40% = $1,500/ $3,750. Lisam's sales ($2,500) would be added to Merick’s sales ($3,750) for total sales of $6,250. Similarly, Lisam's gross profit ($900) would be added to Merick's gross profit ($1,500) for total gross profit of $2,400. Gross profit margin would be 38.4% = ($2,400/$6,250). 50% of Lisam's sales ($1,250) would be added to Merick's sales ($3,750) for total sales of $5,000. Similarly, 50% of Lisam’s gross profit ($450) would be added to Merick’s gross profit of $1,500 for total gross profit of $1,950. Gross profit margin would be 39.0% =

By accessing this mock exam, you agree to the following terms of use: This mock exam is provided to currently-registered CFA candidates. Candidates may view and print the exam for personal exam preparation only. The following activities are strictly prohibited and may result in disciplinary and/or legal action: accessing or permitting access by anyone other than currently-registered CFA candidates; copying, posting to any website, emailing, distributing and/or reprinting the mock exam for any purpose.

($1,950/$5,000). The answer may also be obtained without computations by examining the common size data. Because the parent has a higher gross profit margin than the subsidiary, the consolidation and proportionate consolidation methods would both have a blended margin below 40.0%. 35. After incorporating Lisam’s projected results Merick’s times interest earned for the year ending 31 December 2009 will most likely be highest under: A. equity. B. consolidation. C. proportionate consolidation. Answer: B “Intercorporate Investments,” Susan Perry Williams 2010 Modular Level II, Vol. 2, pp. 32-35, 42-43 Study Session 5-21-c Analyze the effects on financial ratios of the different methods used to account for intercorporate investments. Time interest earned is higher if they use consolidation when compared to proportionate consolidation, and lowest under the equity method.

EBIT(operating income) Interest expense Time interest earned

Merick (Equity)

Lisam

Consolidation

$375

$240

$615

$75 5.00

$30 8.00

$105 5.86

Proportionate Consolidation (only 50% of Lisam added) 495 90 5.50

36. If Merick prepared a consolidated balance sheet on the date of acquisition the total shareholders’ equity ($) under U.S. GAAP will be closest to: A. 1,300. B. 1,800. C. 2,300. Answer: B “Intercorporate Investments,” Susan Perry Williams 2010 Modular Level II, Vol. 2, pp. 43-44 By accessing this mock exam, you agree to the following terms of use: This mock exam is provided to currently-registered CFA candidates. Candidates may view and print the exam for personal exam preparation only. The following activities are strictly prohibited and may result in disciplinary and/or legal action: accessing or permitting access by anyone other than currently-registered CFA candidates; copying, posting to any website, emailing, distributing and/or reprinting the mock exam for any purpose.

Study Session 5-21-a Describe the classification, measurement, and disclosure under the International Financial Reporting Standards (IFRS) for 1) investments in financial assets, 2) investments in associates, 3) joint ventures, 4) business combinations, and 5) special purpose and variable interest entities (SPEs and VIEs). The total consolidated shareholders’ equity would include the shareholders’ equity for Merick (1,300) and the non-controlling interest for Lisam 0.50 x 1,000 = 500. 1,300 + 500 = 1,800. The NCI is now reported in the shareholders equity section under both U.S. GAAP and IFRS.

By accessing this mock exam, you agree to the following terms of use: This mock exam is provided to currently-registered CFA candidates. Candidates may view and print the exam for personal exam preparation only. The following activities are strictly prohibited and may result in disciplinary and/or legal action: accessing or permitting access by anyone other than currently-registered CFA candidates; copying, posting to any website, emailing, distributing and/or reprinting the mock exam for any purpose.

Galaxy Electronics Case Scenario Galaxy Electronics Ltd. (Galaxy) is a manufacturer and distributor of personal computers and hand-held electronic personal organizers. The company had grown rapidly from its inception in 2004 to 2008 but in early 2009, sales growth has slowed significantly and Nadeen Bhatty, the VP Finance, thought that it was a good time for Galaxy to review its accounting methods and ensure they were appropriate for an established company. As a public company they are required to prepare their financial statements in accordance with U.S. GAAP. To this end, the company made the following changes in its accounting methods and estimates in 2009: ● Galaxy produces the computers and organizers based on orders received. A 25% deposit is required on all orders and then Galaxy manufactures and usually ships the units in 2 to 6 weeks. Some orders are placed even further in advance, while some shipments may not occur for up to 3 months following an order. Galaxy had been recording a sale when the product was shipped but now that they were more established, Bhatty changed the revenue recognition point to when the deposits were received. “If the products are made to order, then the critical event is when we receive the order,” she explained. As at August 31, 2009, they had received deposits of $3 million for orders yet to be shipped. The company provides a one-year warranty on their products and records it as a selling and administrative expense at the time of sale. Now, after five years experience with the products, they realized that the actual claims made have been less than the amounts they were accruing. In 2009, the related warranty accounts were adjusted to reflect the new estimated rates. On September 1, 2009, as a result of competitive pressures in the labor market and in recognition of their outstanding work in recent years, the company introduced a restricted stock grant program to all employees who had worked for the company for three years or more. The fair value of the stock on the grant date was $4.2 million; the employee had to remain with the company for 3 years for the shares to vest. While the average volatility of the company’s stock had been in the 38%-42% range in the past three years, with the recent decline in growth that the firm was experiencing, the stock’s volatility had declined to the 19%-24% range. Comparative income statements and balance sheets for Galaxy over the past few years are in Exhibit 1.

By accessing this mock exam, you agree to the following terms of use: This mock exam is provided to currently-registered CFA candidates. Candidates may view and print the exam for personal exam preparation only. The following activities are strictly prohibited and may result in disciplinary and/or legal action: accessing or permitting access by anyone other than currently-registered CFA candidates; copying, posting to any website, emailing, distributing and/or reprinting the mock exam for any purpose.

Exhibit 1 Galaxy Electronics Ltd. (U.S. $ thousands) Income Statement for the year ended August 31st 2009 2008 2007 $100,000 $ 95,000 $ 65,000 47,000 47,500 33,800 53,000 47,500 31,200 34,000 38,000 28,000 2,400 2,700 3,000 16,600 6,800 200 5,478 2,244 67 $ 11,122 $ 4,556 $ 134

Sales Cost of goods sold Gross profit Operating expenses Interest expense Earnings before taxes Income taxes 33% Net Income

Balance Sheet as at August 31st 2009 Assets Cash & investments Accounts receivable Inventories Prepaids and deferrals Total current assets Equipment, net Intangibles Total Assets

2008

$ 21,122 $ 25,000 25,000 13,500 9,000 6,500 4,000 2,000 $ 59,122 $ 47,000 51,000 55,000 21,000 25,000 $131,122 $127,000

Liabilities Accounts payable $ 15,000 $ 11,000 Unearned revenue 4,000 Warranty provision 2,000 4,000 Current portion of long term debt 5,000 5,000 Total current liabilities $ 22,000 $ 24,000 Long term debt 35,000 40,000 Total liabilities $ 57,000 $ 64,000 Shareholders’ equity Common stock 58,000 58,000 Retained earnings 16,122 5,000 $ 74,122 $ 63,000 $131,122 $127,000 Total liabilities & equity

By accessing this mock exam, you agree to the following terms of use: This mock exam is provided to currently-registered CFA candidates. Candidates may view and print the exam for personal exam preparation only. The following activities are strictly prohibited and may result in disciplinary and/or legal action: accessing or permitting access by anyone other than currently-registered CFA candidates; copying, posting to any website, emailing, distributing and/or reprinting the mock exam for any purpose.

37. Which of the following is most likely to be a warning sign of deteriorating earnings quality? The new policy relating to: A. warranty expenses. B. revenue recognition. C. compensation using stock grants. Answer: B “Evaluating Financial Reporting Quality,” Scott Richardson and Irem Tuna 2010 Modular Level II, Vol. 2 pp. 247-251, 301 Study Session 7-25-f Discuss problems with the quality of financial reporting, including revenue recognition, expense recognition, balance sheet issues and cash flow statement issues, and interpret warning signs of these potential problems. The change in revenue recognition to an earlier point, before the product has been produced or delivered, is an aggressive accounting policy that would lower the company’s quality of earnings. 38. The change in estimate for Galaxy’s warranty expense will most likely result in a(n): A. increase in revenue. B. reserve released into income. C. reduction in an off-balance sheet liability. Answer: B “Evaluating Financial Reporting Quality,” Scott Richardson and Irem Tuna 2010 Modular Level II, Vol. 2 pp. 271-275 Study Session 7-25-a, d, f Contrast cash-basis and accrual-basis accounting and explain why accounting discretion exists in the accrual accounting system. Discuss earnings quality and the measures of earnings quality, and compare and contrast the earnings quality of peer companies. Discuss problems with the quality of financial reporting, including revenue recognition, expense recognition, balance sheet issues, and cash flow statement issues, and interpret warning signs of these potential problems. Actual experience has shown that the amount expensed in prior years was too large hence understating net income and effectively creating a reserve. The reversal of the reserve in 2009 would reduce the expense and the liability for 2009 (as seen on the balance sheet) and therefore increase net income. By accessing this mock exam, you agree to the following terms of use: This mock exam is provided to currently-registered CFA candidates. Candidates may view and print the exam for personal exam preparation only. The following activities are strictly prohibited and may result in disciplinary and/or legal action: accessing or permitting access by anyone other than currently-registered CFA candidates; copying, posting to any website, emailing, distributing and/or reprinting the mock exam for any purpose.

39. If an analyst were to adjust Galaxy’s financial statements in an attempt to get a better indication of the company’s revenues, gross profit ($-millions) would most likely decrease by: A. 1.6. B. 4.8. C. 6.4. Answer: C “The Lessons We Learn,” Pamela P. Peterson, CFA and Frank J. Fabozzi, CFA 2010 Modular Level II, Vol. 2 pp. 230-232 “Evaluating Financial Reporting Quality,” Scott Richardson and Irem Tuna 2010 Modular Level II, Vol. 2 pp. 294-295, 288 “Integration of Financial Statement Analysis Techniques,” Jack T. Ciesielski Jr., CFA 2010 Modular Level II, Vol. 2 pp. 316-317 Study Session 7-24-a, 7-25-f, 7-26-e Distinguish among the various definitions of earnings (e.g., EBITDA, operating earnings, net income, etc.). Discuss problems with the quality of financial reporting, including revenue recognition, expense recognition, balance sheet issues, and cash flow statement issues, and interpret warning signs of these potential problems. Analyze and interpret the effects of balance sheet modifications, earnings normalization, and cash-flow-statement-related modifications on a company’s financial statements, financial ratios, and overall financial condition. The firm has received $3 million in deposits. A deposit is normally 25%; hence $12 million has been recognized too soon (no recognition should occur until shipment). With a gross profit margin of 53%, the total adjustment to gross profit would be a decrease of 0.53 x $12 = $6.4. 40. The balance sheet aggregate accruals ($) in the past two years, prior to any adjustments, is closest to: A. 10,000. B. 10,122. C. 14,122. Answer: A “Evaluating Financial Reporting Quality,” Scott Richardson and Irem Tuna 2010 Modular Level II, Vol. 2 pp. 254-257

By accessing this mock exam, you agree to the following terms of use: This mock exam is provided to currently-registered CFA candidates. Candidates may view and print the exam for personal exam preparation only. The following activities are strictly prohibited and may result in disciplinary and/or legal action: accessing or permitting access by anyone other than currently-registered CFA candidates; copying, posting to any website, emailing, distributing and/or reprinting the mock exam for any purpose.

Total Assets Cash & Investments Operating Assets (A) Total Liabilities Long term debt Current portion of LTD Operating Liabilities (B)

2009 ($) 131,122 21,122 110,000

2008 ($) 127,000 25,500 102,000

57,000 35,000 5,000 17,000

64,000 40,000 5,000 19,000

Net operating assets (A-B) 93,000 83,000 Balance sheet based aggregate accruals: change in 10,000 net operating assets “Integration of Financial Statement Analysis Techniques,” Jack T. Ciesielski Jr., CFA 2010 Modular Level II, Vol. 2 pp. 335-337 Study Session 7-25-d, 7-26-c Discuss earnings quality and the measures of earnings quality, and compare and contrast the earnings quality of peer companies. Evaluate the quality of a company’s financial data and recommend appropriate adjustments to improve quality and comparability with similar companies, including adjustments for differences in accounting rules, methods and assumptions. 41. The 2010 stock-based compensation expense ($ millions) will be closest to: A. 0.0. B. 1.4. C. 4.2. Answer: B “Employee Compensation Post-Employment and Share Based,” Elaine Henry, CFA and Elizabeth A. Gordon 2010 Modular Level II, Vol. 2, p. 126 Study Session 6-22-j Explain the impact on financial statements of accounting for stock grants and stock options, and the importance of companies’ assumptions in valuing these grants and options. The compensation expense for restricted stock grants is the fair market value of the shares on the grant date and this amount is allocated over the vesting period: $4.2 million/3 = $1.4 million By accessing this mock exam, you agree to the following terms of use: This mock exam is provided to currently-registered CFA candidates. Candidates may view and print the exam for personal exam preparation only. The following activities are strictly prohibited and may result in disciplinary and/or legal action: accessing or permitting access by anyone other than currently-registered CFA candidates; copying, posting to any website, emailing, distributing and/or reprinting the mock exam for any purpose.

42. The recent change in the volatility of the company’s stock most likely made the cost of the stock compensation program: A. lower. B. higher. C. the same. Answer: C “Employee Compensation Post-Employment and Share Based,” Elaine Henry, CFA and Elizabeth A. Gordon 2010 Modular Level II, Vol. 2, p. 126 Study Session 6-22-j Explain the impact on financial statements of accounting for stock grants and stock options, and the importance of companies’ assumptions in valuing these grants and options. Stock options are affected by higher volatility and would have increased the compensation expense and lowered net income. Stock grants are based on the fair market value of the stock on the day of the grant and are not affected by the stock’s volatility.

By accessing this mock exam, you agree to the following terms of use: This mock exam is provided to currently-registered CFA candidates. Candidates may view and print the exam for personal exam preparation only. The following activities are strictly prohibited and may result in disciplinary and/or legal action: accessing or permitting access by anyone other than currently-registered CFA candidates; copying, posting to any website, emailing, distributing and/or reprinting the mock exam for any purpose.

Scott Case Scenario Cindy Scott is reviewing cash flow projections for a $300,000 capital investment for adaptable equipment to service her company’s manufacturing efforts. After careful study, analysts have determined that when put to the best use over the next five years, the incremental contribution of the equipment produces a positive net present value assuming a 15% annual discount rate (see Exhibit 1). Exhibit 1: Forecasted Cash Flow (in $) Year 1: Sales

Year 2: 425,500

Year 3: 510,600

Year 4: 663,780

Year 5: 531,024

212,750

255,300

331,890

265,512

50,000

50,000

50,000

50,000

60,000

60,000

60,000

60,000

102,750

145,300

221,890

155,512

41,100

58,120

88,756

62,205

61,650

87,180

133,134

93,307

121,650

147,180

193,134

153,307

193,134

20,000 12,000 165,307

370,000 Variable Cash Expenses 185,000 Fixed Cash Expenses 30,000 Depreciation* 60,000 Operating Income Before Tax 95,000 Tax (40%) 38,000 Operating Income After Tax 57,000 After Tax Operating Cash Flow 117,000 Salvage Value Salvage Value After Tax Total After Tax Cash Flow

121,650

147,180

117,000 *

Straight-line over five years. NPV (15% annual discount rate): $183,109 Scott receives a request from her manager, Pat Stevens, to change the cash flows from nominal to real. She decides to remove inflation effects from the sales, variable cost, and salvage value figures, but not to adjust either fixed costs or depreciation. Stevens also requests Scott to calculate both economic and accounting income using the cash flow analysis in Exhibit 1. Scott learns that the equipment is to be financed entirely with a loan at 12%, with interest paid annually for five years and the full principal paid at the end of the fifth year. By accessing this mock exam, you agree to the following terms of use: This mock exam is provided to currently-registered CFA candidates. Candidates may view and print the exam for personal exam preparation only. The following activities are strictly prohibited and may result in disciplinary and/or legal action: accessing or permitting access by anyone other than currently-registered CFA candidates; copying, posting to any website, emailing, distributing and/or reprinting the mock exam for any purpose.

Scott asks another co-worker, Ted Ludlow, for additional suggestions about the analysis. Ludlow makes the following two suggestions: Consider the analysis in Exhibit 1 as a base case and then produce two additional analyses, an optimistic and a pessimistic case, assuming different possible economic environments. Produce these different analyses with a 5-year MACRS depreciation schedule (Exhibit 2). Scott thanks Ludlow for these two suggestions. However, before leaving, Ludlow makes a third suggestion to also calculate operating income after tax less the dollar cost of capital (i.e. the weighted average cost of capital multiplied by the capital investment). Exhibit 2 5-Year MACRS* Schedule Year 1 Year 2 Year 3 Year 4 Year 5 Year 6 20.00% 32.00% 19.20% 11.52% 11.52% 5.76% *MACRS: Modified Accelerated Cost Recovery System for accelerated deprecation

43. Scott’s inflation adjustments to depreciation and fixed costs are most likely: A. correct. B. incorrect because both should be adjusted. C. incorrect because only fixed costs should be adjusted. Answer: B “Capital Budgeting,” John D. Stowe, CFA and Jacques R. Gagne, CFA 2010 Modular Level II, Vol.3, p. 39 Study Session 8-27-b Discuss the effects of inflation on capital budgeting analysis. Both fixed costs and depreciation should be inflation adjusted because inflation reduces the value of tax savings including depreciation. 44. The economic income ($) for Year 3 is closest to: A. 48,365. B. 57,407. C. 109,877. Answer: B “Capital Budgeting,” John D. Stowe, CFA and Jacques R. Gagne, CFA By accessing this mock exam, you agree to the following terms of use: This mock exam is provided to currently-registered CFA candidates. Candidates may view and print the exam for personal exam preparation only. The following activities are strictly prohibited and may result in disciplinary and/or legal action: accessing or permitting access by anyone other than currently-registered CFA candidates; copying, posting to any website, emailing, distributing and/or reprinting the mock exam for any purpose.

2010 Modular Level II, Vol.3, pp. 61-63 Study Session 8-27-h Calculate and interpret accounting income and economic income in the context of capital budgeting. Economic profit = change in market value plus the after-tax cash flow. Beginning Market Value = Present Value of Future Expected After-tax cash flows 147,180 ÷ (1.15)1 + 193,134 ÷ (1.15)2 + 165,307 ÷ (1.15)3 = 382,712 Ending Market Value= Present Value of Future Expected After-tax cash flows 193,134 ÷ (1.15)1 + 165,307 ÷ (1.15)2 = 292,939 Economic Profit: (292,939 – 382,712) + 147,180 = 57,407 See Table 29 on page 61. 45. The accounting income ($) for Year 2 is closest to: A. 25,650. B. 40,050. C. 61,650. Answer: B “Capital Budgeting,” John D. Stowe, CFA and Jacques R. Gagne, CFA 2010 Modular Level II, Vol.3, pp. 61-63 Study Session 8-27-h Calculate and interpret accounting income and economic income in the context of capital budgeting. Operating income before tax less interest equals taxable income: 102,750 – (300,000* 0.12) = 66,750 Accounting income or net income = taxable income x (1 – Tax Rate) 66,750*(1 – 0.40) = 40,050 46. Ted Ludlow’s first suggestion is best described as an example of: A. scenario analysis. B. sensitivity analysis. C. Monte Carlo simulation. Answer: A “Capital Budgeting,” John D. Stowe, CFA and Jacques R. Gagne, CFA 2010 Modular Level II, Vol.3, pp. 44-50 Study Session 8-27-d Explain how sensitivity analysis, scenario analysis, and Monte Carlo simulation can be used to assess the stand-alone risk of a capital project. By accessing this mock exam, you agree to the following terms of use: This mock exam is provided to currently-registered CFA candidates. Candidates may view and print the exam for personal exam preparation only. The following activities are strictly prohibited and may result in disciplinary and/or legal action: accessing or permitting access by anyone other than currently-registered CFA candidates; copying, posting to any website, emailing, distributing and/or reprinting the mock exam for any purpose.

Ludlow’s suggestion of considering alternate economic environments is an example of scenario analysis. 47. If Ludow’s suggestion of using the MACRS depreciation schedule is implemented, the first year’s after-tax operating cash flow will most likely: A. increase. B. decrease. C. remain unchanged. Answer: C “Capital Budgeting,” John D. Stowe, CFA and Jacques R. Gagne, CFA 2010 Modular Level II, Vol.3, p. 33 Study Session 8-27-a Compute the yearly cash flows of an expansion capital project and a replacement capital project and evaluate how the choice of depreciation method affects those cash flows. Because the schedule for the first year is equivalent to straight-line depreciation (1/5 = 20%), the after-tax operating cash flow does not change under straight line or MACRs accelerated depreciation. 48. Ludlow’s third suggestion is best described as the calculation of: A. residual income. B. economic profit. C. free cash flow to equity. Answer: B “Capital Budgeting,” John D. Stowe, CFA and Jacques R. Gagne, CFA 2010 Modular Level II, Vol.3, pp. 64-67 Study Session 8-27-i Differentiate among and evaluate a capital project using the following valuation models: economic profit, residual income, and claims valuation. Ludlow’s suggestion is an example of economic profit (EBIT x (1 – tax rate) $WACC). See page 64.

By accessing this mock exam, you agree to the following terms of use: This mock exam is provided to currently-registered CFA candidates. Candidates may view and print the exam for personal exam preparation only. The following activities are strictly prohibited and may result in disciplinary and/or legal action: accessing or permitting access by anyone other than currently-registered CFA candidates; copying, posting to any website, emailing, distributing and/or reprinting the mock exam for any purpose.

Louise Tremblay Case Scenario Louise Tremblay, CFA, is a portfolio manager for a global equity fund domiciled in the United States. She wants to add positions in foreign stocks of Canada and Brazil, two countries where there is currently no exposure. Tremblay places a call to Hal Baroque, the firm’s economist, to arrange a meeting to discuss both his outlook for these economies and some issues related to foreign exchange relations and international asset pricing. During the meeting, Baroque presents the information he gathered in preparation for their discussion, as shown below in Exhibit 1. Exhibit 1 Selected Currency Exchanges and Market Rates One Year Expected Country Currency Spot Exchange Rate* Risk-free Rate Annual Inflation Rate U.S. USD$ NA 4.80% 2.30% Canada CAD$ 1.2138-1.2259 4.10% 1.90% Brazil Real 2.3844-2.4082 8.80% 6.30 % *Number of foreign currency units per one U.S. dollar Baroque begins his discussion by reviewing some basic relations that are useful in understanding the interplay between exchange rates, interest rates, and inflation. He remarks, “Theoretically, the interest rate differential between two countries should be equal to the expected inflation rate differential over the term of the interest rate.” Tremblay provides two justifications for adding Canadian stocks to her portfolio: 1) The real returns are currently higher in Canada than in the United States. 2) By her own prediction, Canada will experience higher economic growth than the United States over the next three to five years. She is convinced that changes in Canada’s financial account brought about by this higher growth will dominate any changes in Canada’s current account that might occur because of this higher relative growth. Baroque’s outlook for Brazil’s economy is not favorable. He explains that in general, the economy of an emerging market tends to have a strongly positive correlation with the value of both its currency and its stock markets. Baroque thinks that Brazil’s economic situation will continue to deteriorate along with the value of its currency, which he expects to depreciate by 5 percent against the U.S. dollar over the coming year.

49. Given a bid-side quote on the three-month forward contract of CAN$1.1986 per U.S. dollar, the three-month forward U.S. dollar is quoted at an annualized: A. 5.00% discount. B. 1.25% discount. C. 5.00% premium. By accessing this mock exam, you agree to the following terms of use: This mock exam is provided to currently-registered CFA candidates. Candidates may view and print the exam for personal exam preparation only. The following activities are strictly prohibited and may result in disciplinary and/or legal action: accessing or permitting access by anyone other than currently-registered CFA candidates; copying, posting to any website, emailing, distributing and/or reprinting the mock exam for any purpose.

Answer: A “Currency Exchange Rates,” Bruno Solnik and Dennis McLeavey, CFA 2010 Modular Level II, Vol. 1, pp. 596-598 Study Session 4-18-i Distinguish between spot and forward transactions, and calculate the annualized forward premium/discount for a given currency and infer whether the currency is “strong” or “weak.” The U.S dollar buys less Canadian dollars in the future: Premium or Discount = [(forward rate – spot rate)/(spot rate)] x (12 / # of months) x 100% = (1.1986 – 1.2138)/(1.2138)) x (12/3) x 100% = -0.0152/1.2138 x 4 x 100% = -0.0125 x 4 x 100% = -5.00% 50. According to purchasing power parity and based on the spot rate bid quote, the one-year forward exchange rate for the Canadian dollar per U.S. dollar is closest to: A. CAD$1.2057. B. CAD$1.2091. C. CAD$1.2186. Answer: B “Foreign Exchange Parity Relations,” Bruno Solnik and Dennis McLeavey, CFA 2010 Modular Level II, Vol. 1, pp. 628-629 Study Session 4-19-g, h Define and discuss absolute purchasing power parity and relative purchasing power parity. Calculate the end-of-period exchange rate implied by purchasing power parity, given the beginning-of-period exchange rate and the inflation rates. Purchasing power parity can be written as S1/S0 = (1+IFC)/(1+IDC). In this case, S1/1.2138 = (1.019)/(1.023) = 0.9961. S1 = 1.2091 51. If a dealer’s bid-side quote for the Canadian Dollar/Brazilian Real is CAD$0.5250, Tremblay’s profit on a USD$1,000,000 initial investment in the triangular arbitrage opportunity is closest to: A. USD$21,135. B. USD$31,315. C. USD$31,328. By accessing this mock exam, you agree to the following terms of use: This mock exam is provided to currently-registered CFA candidates. Candidates may view and print the exam for personal exam preparation only. The following activities are strictly prohibited and may result in disciplinary and/or legal action: accessing or permitting access by anyone other than currently-registered CFA candidates; copying, posting to any website, emailing, distributing and/or reprinting the mock exam for any purpose.

Answer: A “Currency Exchange Rates,” Bruno Solnik and Dennis McLeavey, CFA 2010 Modular Level II, Vol. 1, pp. 595-596 Study Session 4-18-d Calculate the profit on a triangular arbitrage opportunity, given the bid-ask quotations for the currencies of the three countries involved in the arbitrage. It is cheaper to buy Canadian dollars indirectly through Brazilian Reals than directly with U.S. Dollars. This creates a triangular arbitrage opportunity: USD$1,000,000 x (2.3844) = 2,384,400 Brazilian Real 2,384,400 x (.5250) = CAD$1,251,810 CAD$1,251,810/1.2259 = USD$1,021,135 USD$1,021,135 – USD$1,000,000 = USD$21,135 profit 52. The specific relation referred to in Baroque’s remark most accurately describes the: A. interest rate parity relation. B. international Fisher relation. C. purchasing power parity relation. Answer: B “Foreign Exchange Parity Relations,” Bruno Solnik and Dennis McLeavey, CFA 2010 Modular Level II, Vol. 1, pp. 630-632 Study Session 4-19-i, k Define and discuss the international Fisher relation. Calculate the international Fisher relation, and its linear approximation between interest rates and expected inflation rates. The international Fisher relation states that the interest rate differential between two countries should be equal to the expected inflation rate differential over the term of the interest rate. 53. Which of Tremblay’s justifications for adding Canadian stocks to the portfolio is most appropriate? A. 1 only B. 2 only C. Both 1 and 2 Answer: C By accessing this mock exam, you agree to the following terms of use: This mock exam is provided to currently-registered CFA candidates. Candidates may view and print the exam for personal exam preparation only. The following activities are strictly prohibited and may result in disciplinary and/or legal action: accessing or permitting access by anyone other than currently-registered CFA candidates; copying, posting to any website, emailing, distributing and/or reprinting the mock exam for any purpose.

“Foreign Exchange Parity Relations,” Bruno Solnik and Dennis McLeavey, CFA 2010 Modular Level II, Vol. 1, pp. 618-623 Study Session 4-19-d Describe the factors that cause a nation’s currency to appreciate or depreciate. Higher real rates and higher growth are both appropriate justifications for investing in Canada. 54. Based on the data presented in Exhibit 1, Baroque’s expectation for Brazil’s currency is best described as: A. interest rate parity. B. purchasing power parity C. uncovered interest rate parity. Answer: C “Foreign Exchange Parity Relations,” Bruno Solnik and Dennis McLeavey, CFA 2010 Modular Level II, Vol. 1, pp. 630-633 Study Session: 4-19-g, h, l, m Define and discuss absolute purchasing power parity and relative purchasing power parity. Calculate the end-of-period exchange rate implied by purchasing power parity, given the beginning-of-period exchange rate and the inflation rates. Define and discuss the theory of uncovered interest rate parity, and explain the theory’s relationship to other exchange rate parity theories. Calculate the expected change in the exchange rate, given interest rates and the assumption that uncovered interest rate parity holds. Uncovered interest rate parity is an economic theory about expectations. Baroque is stating an expectation regarding Brazil’s currency. Interest rate parity allows one to price a forward exchange rate contract given the spot exchange rate and the two interest rates. Baroque is not pricing a forward contract; rather, he is expressing an expectation. For the data in Exhibit 1, interest rate parity implies an approximate 4 percent depreciation in the Real against the USD. Baroque expects a 5 percent depreciation, which is not consistent with interest rate parity. For the data in Exhibit 1, purchasing power parity (PPP) implies an expected spot rate of 2.47763 (calculated as S1/2.3844 = 1.063/1.023; solve for S1). Baroque expects the currency to depreciate 5% - 2.3844(1.05) = 2.5036 which is not consistent with the S1 value calculated with PPP.

By accessing this mock exam, you agree to the following terms of use: This mock exam is provided to currently-registered CFA candidates. Candidates may view and print the exam for personal exam preparation only. The following activities are strictly prohibited and may result in disciplinary and/or legal action: accessing or permitting access by anyone other than currently-registered CFA candidates; copying, posting to any website, emailing, distributing and/or reprinting the mock exam for any purpose.

Chan Mei Yee Case Scenario Chan Mei Yee is valuing McLaughlin Corporation common shares using a free cash flow approach. She assembled information about McLaughlin from several sources. She begins her analysis by determining free cash flow to the firm (FCFF) and free cash flow to equity (FCFE) for the 2009 fiscal year, using the financial statements in Exhibits 1 and 2 and other financial information contained in Exhibit 3. McLaughlin’s fiscal year ends 31 December. Chan plans to perform two different valuations of McLaughlin, which she calls the “base case” valuation and the “alternative” valuation. Critical assumptions for each are given below and in Exhibit 3. Base case valuation 2010 FCFF will be $600 million. FCFF will grow forever at 4 percent annually. The market value and book value of McLaughlin’s long-term debt are approximately equal. Alternative valuation 2010 earnings per share (EPS) will be $1.80. EPS will grow forever at 6 percent annually. For 2010 and beyond: o Net capital expenditures (fixed capital expenditures minus depreciation) will be 30 percent of EPS. o Investments in working capital will be 10 percent of EPS. o 60 percent of future investments will be financed with equity and 40 percent will be financed with debt. Chan is also concerned about the effects on McLaughlin’s 2010 FCFE of the following three possible financial actions by McLaughlin during the year 2010. Increasing common stock cash dividends by $110 million. Repurchasing $60 million of common shares. Reducing its outstanding long-term debt by $100 million. Melissa Nicosia, Chan’s supervisor, reviews McLaughlin’s valuations. Specifically, Nicosia makes the following two statements: 1) The free cash flow valuation approach is superior to the discounted dividend valuation approach because the company’s dividends have been substantially different from its FCFE, and 2) Because the company’s capital structure seems unstable, the FCFE valuation approach is superior to the FCFF valuation approach.

By accessing this mock exam, you agree to the following terms of use: This mock exam is provided to currently-registered CFA candidates. Candidates may view and print the exam for personal exam preparation only. The following activities are strictly prohibited and may result in disciplinary and/or legal action: accessing or permitting access by anyone other than currently-registered CFA candidates; copying, posting to any website, emailing, distributing and/or reprinting the mock exam for any purpose.

Exhibit 1 McLaughlin Corporation Selected Financial Data (in millions, except per share amounts) For Year Ending 31 December

2009 $ 6,456 3,363 1,744 1,349

Revenues Cost of goods sold Selling, general, and administrative expense Earnings before interest, taxes, depreciation, and amortization (EBITDA) Depreciation expense Operating income Interest expense Pretax income Income tax Net income

243 1,106 186 920 294 $ 626

Number of outstanding shares (millions) 2009 Earnings per share 2009 Dividends paid (millions) 2009 Dividends per share 2009 Fixed capital expenditures (millions)

$ $ $ $

411 1.52 148 0.36 535

Exhibit 2 McLaughlin Corporation Consolidated Balance Sheets (in millions)

Cash and cash equivalents Accounts receivable Inventories Other current assets Total current assets Long-term assets, net Total assets

at 31 December 2009 2008 $ 32 $ 21 413 417 709 638 136 123 1,290 1,199 4,814 4,522 $ 6,104 $ 5,721

Current liabilities Long-term debt Common stockholders’ equity Total liabilities and stockholders’ equity

$ 2,783 2,249 1,072 $ 6,104

$ 2,678 2,449 594 $ 5,721

By accessing this mock exam, you agree to the following terms of use: This mock exam is provided to currently-registered CFA candidates. Candidates may view and print the exam for personal exam preparation only. The following activities are strictly prohibited and may result in disciplinary and/or legal action: accessing or permitting access by anyone other than currently-registered CFA candidates; copying, posting to any website, emailing, distributing and/or reprinting the mock exam for any purpose.

Exhibit 3 Other Current Financial Information for McLaughlin Corp. Effective tax rate 32.0% Cost of equity 12.0% Weighted average cost of capital 9.0% Non-operating assets $0 55. McLaughlin’s FCFF ($ millions) for 2009 is closest to: A. 418. B. 460. C. 485. Answer: C "Free Cash Flow Valuation," by Jerald Pinto, CFA, Elaine Henry, CFA, Thomas Robinson, CFA, and John Stowe, CFA 2010 Modular Level II, Vol. 4, pp. 384-387 Study Session 12-41-d, e Discuss the appropriate adjustments to net income, earnings before interest and taxes (EBIT), earnings before interest, taxes, depreciation, and amortization (EBITDA), and cash flow from operations (CFO) to calculate FCFF and FCFE. Calculate FCFF and FCFE when given a company’s financial statements prepared according to International Financial Reporting Standards (IFRS) or U.S. generally accepted accounting principles (GAAP). FCFF = NI + NCC + Int(1 – Tax Rate) – FCInv – WCInv Net income (given) = $626; Interest Expense (given) = $186; Tax rate = 294/920 = 32% Non cash charges (depreciation) (given) = $243; Fixed capital expenditures (given) = $535 WC Investment Current assets excluding cash Current liabilities Working capital

2009 ($) 1,290 - 32 = 1,258 2,783 -1,525

2008 ($)

Net increase ($)

1,199 - 21 =1,178 2,678 -1,500

-25

FCFF = 626 + 243 + 186(1 – 0.32) – 535 - (-25) = 485.48 = $485 million

By accessing this mock exam, you agree to the following terms of use: This mock exam is provided to currently-registered CFA candidates. Candidates may view and print the exam for personal exam preparation only. The following activities are strictly prohibited and may result in disciplinary and/or legal action: accessing or permitting access by anyone other than currently-registered CFA candidates; copying, posting to any website, emailing, distributing and/or reprinting the mock exam for any purpose.

56. The estimate of 2009 FCFE ($ millions) is closest to: A. -327. B. 159. C. 509. Answer: B "Free Cash Flow Valuation," by Jerald Pinto, CFA, Elaine Henry, CFA, Thomas Robinson, CFA, and John Stowe, CFA 2010 Modular Level II, Vol. 4, pp. 396-397; 400-401 Study Session 12-41- e Calculate FCFF and FCFE when given a company’s financial statements prepared according to International Financial Reporting Standards (IFRS) or U.S. generally accepted accounting principles (GAAP). FCFE = NI + NCC – FCInv – WCInv + Net Borrowing (see, p. 397) Net income (given) = $626; Non cash charges (depreciation) (given) = $243; Fixed capital expenditures (given) = $535 WC Investment 2009 ($) 2008 ($) Net increase ($) Current assets excluding cash 1,290 - 32 = 1,258 1,199 - 21 =1,178 Current liabilities 2,783 2,678 Working capital -1,525 -1,500 -25 Long-term debt

2,249

2,449

-200

FCFE = 626 + 243 – 535 – (-25) + (-200) = $159 million 57. Using Chan’s base case valuation assumptions and the FCFF valuation approach, the year-end 2009 value per share ($) of McLaughlin common stock is closest to: A. 18.25. B. 23.73. C. 29.20. Answer: B "Free Cash Flow Valuation," by Jerald Pinto, CFA, Elaine Henry, CFA, Thomas Robinson, CFA, and John Stowe, CFA 2010 Modular Level II, Vol. 4, pp. 382-387 Study Session 12-41-k Calculate the value of a company using the stable-growth, two-stage, and threestage FCFF and FCFE models. By accessing this mock exam, you agree to the following terms of use: This mock exam is provided to currently-registered CFA candidates. Candidates may view and print the exam for personal exam preparation only. The following activities are strictly prohibited and may result in disciplinary and/or legal action: accessing or permitting access by anyone other than currently-registered CFA candidates; copying, posting to any website, emailing, distributing and/or reprinting the mock exam for any purpose.

The value of the firm is: FCFF1 600 Firm value $12,000 million. WACC g 0.09 0.04 Equity value = Firm value – Market value of debt = 12,000 – 2,249 = $9,751 million. Value per share = Equity value / Number of shares = 9,751 million / 411 million = 23.7251 = $23.73 per share. 58. Using Chan’s alternative valuation assumptions and the FCFE valuation approach, the year-end 2009 value of McLaughlin’s common stock ($) is closest to: A. 18.00. B. 22.80. C. 24.17. Answer: B "Free Cash Flow Valuation," by Jerald Pinto, CFA, Elaine Henry, CFA, Thomas Robinson, CFA, and John Stowe, CFA 2010 Modular Level II, Vol. 4, pp. 408, 382-383 Study Session 12-41-f, k Discuss approaches for forecasting FCFF and FCFE. Calculate the value of a company using the single-stage, two-stage, and threestage FCFF and FCFE models. FCFE = Net income – (1 – DR) (FCInv – Depreciation) – (1 – DR) (WCInv) FCFE1 = 1.80 – (1 – 0.40) (0.30 ×1.80) – (1 – 0.40) (0.10 ×1.80) FCFE1 = 1.80 – 0.324 – 0.108 = 1.368. FCFE will grow at the same rate as Net income, 6% annually.

FCFE1 FCFE 0 (1 g ) r g r g The value per share is $22.80. Equity value

1.368 0.12 0.06

$22.80

59. The most likely combined effect of the three possible financial actions will reduce McLaughlin’s 2010 FCFE ($ millions) by: A. 100. B. 160. C. 270. Answer: A By accessing this mock exam, you agree to the following terms of use: This mock exam is provided to currently-registered CFA candidates. Candidates may view and print the exam for personal exam preparation only. The following activities are strictly prohibited and may result in disciplinary and/or legal action: accessing or permitting access by anyone other than currently-registered CFA candidates; copying, posting to any website, emailing, distributing and/or reprinting the mock exam for any purpose.

"Free Cash Flow Valuation," by Jerald Pinto, CFA, Elaine Henry, CFA, Thomas Robinson, CFA, and John Stowe, CFA 2010 Modular Level II, Vol. 4, pp. 404-405 Study Session 12-41-h Explain how dividends, share repurchases, share issues, and changes in leverage may affect FCFF and FCFE. The three possible actions are: dividend increase = 110; share repurchase = 60; and the debt repayment = 100. Reducing debt by $100 million reduces FCFE (the amount of cash available to equity holders) by that amount. The cash dividend and the share repurchase are uses of FCFE, and do not change the amount of cash available to equity holders. Therefore FCFE will reduce by $100 million. 60. The most accurate inference one can make in reference to Nicosia’s two statements pertaining to McLaughlin’s valuations is that she is: A. correct with respect to both. B. incorrect with respect to both. C. correct with respect to statement (1) but not (2). Answer: C "Free Cash Flow Valuation," by Jerald Pinto, CFA, Elaine Henry, CFA, Thomas Robinson, CFA, and John Stowe, CFA 2010 Modular Level II, Vol. 4, p. 354, pp. 411-416 Study Session 12-41-c, n Contrast the ownership perspective implicit in the FCFE approach to the ownership perspective implicit in the dividend discount approach. Describe the characteristics of companies for which the FCFF model is preferred to the FCFE model. Analysts should use free cash flow valuation whenever dividends differ significantly from the company’s capacity to pay dividends. FCFF valuation is preferred over FCFE valuation whenever the capital structure is unstable or everchanging. So Chan’s first statement is correct, and her second statement is incorrect.

By accessing this mock exam, you agree to the following terms of use: This mock exam is provided to currently-registered CFA candidates. Candidates may view and print the exam for personal exam preparation only. The following activities are strictly prohibited and may result in disciplinary and/or legal action: accessing or permitting access by anyone other than currently-registered CFA candidates; copying, posting to any website, emailing, distributing and/or reprinting the mock exam for any purpose.